Is the Work Done by a Variable Force Non-Conservative?

In summary: Yeah, you replace it with whatever variable you want.the equation of path is x=(2a) sinθ and y=(a) cosθWhat is ##\theta## here? It can't be the argument parameter of the polar representation of (x,y) since we would deduce ##\tan^2(\theta)=\frac 12##. So I assume it is an arbitrary parameter unrelated to polar coordinates. But then, where do the integration bounds come from?I see no diagram.
  • #36
kuruman said:
LaTeX is preferable, but if your work is too involved, you can post a picture as long as it is clearly legible and right side up. Use the "Attach files" link, lower left.
Thank you ... I have done it with a math editor.
 

Attachments

  • 1121212.png
    1121212.png
    8.8 KB · Views: 64
Physics news on Phys.org
  • #37
haruspex said:
I don't get the factor as 6. Did you handle the signs correctly? The rest looks fine.
What about Theta ?! Did I specify theta correctly?
I get the factor 6 again ! Let me send a picture ...
 
  • #38
haruspex said:
I don't get the factor as 6. Did you handle the signs correctly? The rest looks fine.
 

Attachments

  • 1221.png
    1221.png
    10.9 KB · Views: 62
  • #39
I'm getting what you are getting.

$$ \int 3 x y^2 dy = \int 3 \cdot y^2 \cdot 2 \sqrt{a^2 - y^2} dy $$

Let:

##\frac{y}{a} = \sin \beta##
## \frac{ \sqrt{a^2 - y^2} }{a} = \cos \beta##

It follows that

$$ y^2 = a^2 \sin^2 \beta$$
$$\sqrt{a^2 - y^2} = a \cos \beta$$
$$ dy = a \cos \beta d \beta$$

$$ \implies \int 3 x y^2 dy = 6 a^4 \int \sin^2 \beta \cos^2 \beta d \beta $$
 
Last edited:
  • Like
Likes MatinSAR
  • #40
haruspex said:
I don't get the factor as 6. Did you handle the signs correctly? The rest looks fine.
I also get a factor of 6, although I would never have done the math the way that he did it>
 
  • Like
Likes MatinSAR
  • #41
Chestermiller said:
I also get a factor of 6, although I would never have done the math the way that he did it>
Was it true ?!
What was wrong with the way I have done the math :rolleyes:
erobz said:
I'm getting what you are getting.

Can you please tell me what is your final answer ?!
 
  • #42
MatinSAR said:
Was it true ?!
What was wrong with the way I have done the math :rolleyes:

Can you please tell me what is your final answer ?!
I didn't do the integral!
 
  • Like
Likes MatinSAR
  • #43
Post your work, it will get checked by someone...
 
  • Like
Likes MatinSAR
  • #44
MatinSAR said:
What about Theta ?! Did I specify theta correctly?
I get the factor 6 again ! Let me send a picture ...
Yes, my mistake… I should have waited until I had the time to check properly.
 
  • Like
Likes MatinSAR
  • #45
erobz said:
Post your work, it will get checked by someone...
1668633876399.png
 
  • #46
MatinSAR said:
It is a line integral, not a double integral, so you must arrange that there is only one variable in the integrand. If you want to do it as an integral wrt y then you must first replace all the occurrences of x with what x is as a function of y.
But really, the parametric approach using theta is much easier.
 
  • Like
Likes MatinSAR and erobz
  • #47
  • Like
Likes MatinSAR
  • #48
haruspex said:
It is a line integral, not a double integral, so you must arrange that there is only one variable in the integrand. If you want to do it as an integral wrt y then you must first replace all the occurrences of x with what x is as a function of y.
But really, the parametric approach using theta is much easier.
So my answer is wrong , isn't it ?!
Thank you ... I will try to do it using parametric approach and I will send a picture of the work ...
erobz said:
Show the work. Explain how you get the last result?
Thank you ... I will try to do it using parametrich approach and I will send a picture of the work ...
 
  • Like
Likes erobz
  • #49
MatinSAR said:
I will try to do it using parametric approach
I'm confused. Isn’t that what you already did in post #32? Except that you did not perform the integral.
 
  • Like
Likes MatinSAR
  • #50
The integration is very simple. It just involves the use of trigonometric identities.$$(\sin{\theta}\cos{\theta})^2=\frac{\sin^2{2\theta}}{4}=\frac{1-\cos{4\theta}}{8}$$
 
Last edited:
  • Like
Likes MatinSAR
  • #51
MatinSAR said:
Hi @MatinSAR. Can I point out a fundamental mistake you are making? I don’t think anyone has mentioned it yet (apologies if they already have).

The final expression for W can not contain x or y (or θ). These are variables which change as you move along the path. They must disappear when the definite integral is evaluated.

You made the same mistake in your Post #1 solution, where your final expression for W contained x and y.

The expression for W can contains only given constants (here only the 'a') and pure numbers like 5, π and √2 (just picking random values as examples).

For example, if you determine the formula for the circumference of a circle, x²+y²=a², it wouldn’t make sense to have x and/or y in the final formula.
 
  • Like
Likes MatinSAR
  • #52
haruspex said:
I'm confused. Isn’t that what you already did in post #32? Except that you did not perform the integral.
Yes. It was unfinished.
Chestermiller said:
The integration is very simple. It just involves the use of trigonometric identities.$$(\sin{\theta}\cos{\theta})^2=\frac{\sin^2{2\theta}}{4}=\frac{1-\cos{4\theta}}{8}$$
Thank you.
Steve4Physics said:
Hi @MatinSAR. Can I point out a fundamental mistake you are making? I don’t think anyone has mentioned it yet (apologies if they already have).

The final expression for W can not contain x or y (or θ). These are variables which change as you move along the path. They must disappear when the definite integral is evaluated.

You made the same mistake in your Post #1 solution, where your final expression for W contained x and y.

The expression for W can contains only given constants (here only the 'a') and pure numbers like 5, π and √2 (just picking random values as examples).

For example, if you determine the formula for the circumference of a circle, x²+y²=a², it wouldn’t make sense to have x and/or y in the final formula.
Thank you ... I didn't know about this.
 
  • Like
Likes Steve4Physics
  • #53
I hope it's finally true ...
photo_2022-11-17_10-20-59.jpg
The force is conserative so If I choose another path the final answer shouldn't change, Is it true ?!
 
  • #54
MatinSAR said:
The force is conserative
Is it? What do you get if you integrate from 0 to 2π?

I agree with your answer.
 
  • Like
Likes MatinSAR
  • #55
haruspex said:
Is it?
It was mentioned in question.
haruspex said:
I agree with your answer.
Thank you.
 
  • #56
haruspex said:
.
Steve4Physics said:
.
Chestermiller said:
.
erobz said:
.
kuruman said:
.
Mister T said:
.
I hope I haven't forgotten anyone.

Thank you for your help and time.
🙏🙏🙏
 
  • Like
Likes erobz
  • #57
MatinSAR said:
It was mentioned in question.
Try the path along the axes. Isn't F always zero there?
 
  • Like
Likes MatinSAR
  • #58
haruspex said:
Try the path along the axes. Isn't F always zero there?
No it's not.
 

Attachments

  • 1212.jpg
    1212.jpg
    61.3 KB · Views: 58
  • #59
MatinSAR said:
No it's not.
Oh yes it is! (If you familiar with British pantomime.)

Can I expand on what @haruspex said in Post #57?

Referring to the Post #21 diagram, suppose you take the following route:
- from A(0,a), move along the y-axis to the origin (0,0);
- from the origin (0,0) move along the x-axis to C(2a,0).

##\vec F = x^2y~\hat i + xy^2 \hat~j##

While moving along the y-axis (x=0); both components of ##\vec F## are zero.
Similarly while moving along the x-axis.

The total work done along this path is therefore zero. This is different to your calculated value for the original route from A to C.

So you can see (without doing any maths) that the force is not conservative.
 
  • Like
Likes MatinSAR
  • #60
Steve4Physics said:
Referring to the Post #21 diagram, suppose you take the following route:
- from A(0,a), move along the y-axis to the origin (0,0);
- from the origin (0,0) move along the x-axis to C(2a,0).
Great !
So it's not conserative ...
Thank you.
 

Similar threads

  • Introductory Physics Homework Help
Replies
7
Views
2K
  • Introductory Physics Homework Help
Replies
4
Views
1K
Replies
29
Views
1K
  • Introductory Physics Homework Help
Replies
9
Views
201
Replies
4
Views
1K
  • Introductory Physics Homework Help
Replies
25
Views
1K
  • Introductory Physics Homework Help
Replies
11
Views
1K
  • Introductory Physics Homework Help
2
Replies
57
Views
3K
  • Introductory Physics Homework Help
Replies
8
Views
3K
  • Introductory Physics Homework Help
Replies
12
Views
851
Back
Top